Learn
Look at this equation:
C
c³ = 125
What is c, the cube root of 125?

Answers

Answer 1
C = 5.

5^3 = 5 x 5 x 5 = 125.

I hope this helps!!

Related Questions

2(x-1)+5(x-9) Show your work please!

Answers

Step-by-step explanation:

2(x-1) + 5(x-9)

2x - 2 + 5x - 45

2x + 5x - 2 - 45

7x - 47

Answer:

2x -2 + 5x -45       7x -47

Step-by-step explanation:

2*x = 2x

2*-1 = -2

5*x =5x

5*-9 = -45

An automobile purchased for $28,000 is worth $2500 after 5 years. Assuming that the car's value depreciated steadily from year to year, what was it worth at the end of the third year?
The value 3 years after it was purchased is

Answers

Answer:

$3285.47

Step-by-step explanation:

We know the value decreased by a factor of 2500/28000 after 5 years.

This means the value decreased by a factor of (2500/28000)⅕ per year.

Therefore, after 3 years, the value is

[tex]28000(2500/28000)^{3/5} \approx 3285.47[/tex]

A high school student decides to apply four of six famous colleges. I’m how many possible ways can the four colleges be selected

Answers

If a high school student decides to apply four of six famous colleges, then the four colleges can be selected in as many as 15 different combinations.

As per question statement, a high school student decides to apply four of six famous colleges.

We are required to calculate the number of combinations in which the four colleges can be selected.

To solve this question, we need to know the formula of Combination which goes as [tex](nCr)=\frac{n!}{r!(n-r)!}[/tex]

, i.e., we are to select a set or "r" from a set of "n".

Here, we have to select a combination of 4 from a set of 6.

Therefore applying (nCr) formula with (n = 6) and (r = 4), we get,

[tex](6C4)=\frac{6!}{4!(6-4)!} =\frac{6!}{4!2!}=\frac{4!*5*6}{4!*2}=\frac{5*6}{2}=(5*3)=15[/tex].

Combinations: In mathematics, a combination is a method of selecting items from a particular set, where the order of selection does not matter, i.e., if we have a set of three P, Q and R, then in how many ways we can select two numbers from each set, can be easily defined by combination.

To learn more about combinations, click on the link below.

brainly.com/question/8561440

#SPJ9

2 and 3 50 points and Brainlyest

Answers

AAnswer:

AB (-1, 4) = 34^ and B = (3,-1) 249*32 = 20

Step-by-step explanation:

Answer:

[tex]AB=\sqrt{53}\:\sf units[/tex]

[tex]EF=\sqrt{349}\:\sf units[/tex]

Step-by-step explanation:

Distance between two points

[tex]d=\sqrt{(x_2-x_1)^2+(y_2-y_1)^2}[/tex]

[tex]\textsf{where }(x_1,y_1) \textsf{ and }(x_2,y_2)\:\textsf{are the two points}.[/tex]

Given:

A = (-4, 1)B = (3, -1)

Substitute the given points into the distance formula and solve for AB:

[tex]\begin{aligned}\implies AB & =\sqrt{(x_B-x_A)^2+(y_B-y_A)^2}\\& =\sqrt{(3-(-4))^2+(-1-1)^2}\\& =\sqrt{(7)^2+(-2)^2}\\& =\sqrt{49+4}\\& = \sqrt{53}\end{aligned}[/tex]

Given:

E = (-7, -2)F = (11, 3)

Substitute the given points into the distance formula and solve for EF:

[tex]\begin{aligned}\implies EF & =\sqrt{(x_F-x_E)^2+(y_F-y_E)^2}\\& =\sqrt{(11-(-7))^2+(3-(-2))^2}\\& =\sqrt{18^2+5^2}\\& =\sqrt{324+25}\\& =\sqrt{349}\end{aligned}[/tex]

Learn more about the distance formula here:

https://brainly.com/question/28144723

https://brainly.com/question/28247604

Which measurement is closet to the lateral surface area in square centimeters of the cylinder?

H = 7.6

B = 5.8

Answers

A. 164.9 cm (2)

B. 277.0 cm (2)

C. 138.5 cm (2)

D. 191.3 cm (2)

Answers

The value closest to the lateral area of the cylinder is 138. 5cm^2. Option C

How to determine the value

The formula for determining the lateral area of a cylinder is expressed as;

Lateral area = 2 πrh

Where;

r is the radiush is the height of the cylinder

Note that radius is half the diameter;

radius, r = 5. 8/ 2 = 2. 9 cm

Substitute into the formula

Lateral area = 2 ( 3. 142 × 2. 9 × 7. 6)

Lateral area = 2 ( 69. 249)

Lateral area = 138. 49 cm^2

Thus, the value closest to the lateral area of the cylinder is 138. 5cm^2. Option C

Learn more about lateral area here:

https://brainly.com/question/15777841

#SPJ1

Which is true?
5/4 > 5/6
3/3 > 4/3
3/3 < 3/6

Answers

Answer:

5/4 > 5/6

Step-by-step explanation:

Find the measure of angle X.

Answers

Answer:

x = 23 degrees

Because if you look at diagram x is vertical to angle FBC, and angle FEC and angle EBA are alternate interior angles so they are equal to each other, and AC is a straight line which means it equals to 180 degrees.

[tex]4x+65+x = 180[/tex]

x = 23 degrees

Refer to the photo below. First, construct a segment extending from EB.

What is 5+3 72 times

Answers

Answer: 576

Step-by-step explanation:

5+3 = 8

8*72=576

Answer:576

Step-by-step explanation: just add and multiply

Please answer fast..............................................

Answers

Answer:

[tex]V=lwh\\\frac{V}{lw} = \frac{lwh}{lw} \\\frac{V}{lw} = h[/tex]

If f(x) = 2ax + b/x-1, lim x→0 f(x) = -3, lim x→∞ f(x)=4, prove that: f(2)= 11. ​

Answers

If

[tex]f(x) = \dfrac{2ax+b}{x-1}[/tex]

Note that

[tex]f(2) = \dfrac{4a+b}{2-1} = 4a+b[/tex]

From the given information we have

[tex]\displaystyle \lim_{x\to0} f(x) = \lim_{x\to0} \frac{2ax+b}{x-1} = \frac{0+b}{0-1} = -b = -3 \\\\ ~~~~ \implies b=3[/tex]

and

[tex]\displaystyle \lim_{x\to\infty} f(x) = \lim_{x\to\infty} \frac{2ax+b}{x-1} = \lim_{x\to\infty} \frac{2a+\frac bx}{1-\frac1x} = \frac{2a+0}{1-0} = 2a = 4 \\\\ ~~~~ \implies a=2[/tex]

It follows that

[tex]f(2) = 4\cdot2+3 = 11[/tex]

as required.

Under her cell phone plan, Morgan pays a flat cost of $54 per month and $5 per gigabyte, or part of a gigabyte. (For example, if she used 2.3 gigabytes, she would have to pay for 3 whole gigabytes.) She wants to keep her bill under $70 per month. What is the maximum whole number of gigabytes of data she can use while staying within her budget?

Answers

Answer: She can use 4 gigabyte while remaining in her budget.

Step-by-step explanation:

You would first subtract 54  from 70 which leaves you with 16,

Then you would divide 16 by 4 to find how many gigabytes she could use which gives you 4

So she can use 4 gigabyte while remaining in her budget.

Learn  more about basic mathematics operations here

https://brainly.com/question/28493322

#SPJ9

Answer: Answer is 3

Step-by-step explanation:

let the maximum number of gigabytes of data is x

total budget is  $70

then, flat cost is $54 and $5per gigabyte

we get:

54 +5 × x= 70

5x =70 - 54

x = [tex]\frac{16}{5}[/tex]  = 3.2 ≈ 3

learn more about this type question here :https://brainly.in/question/53477697?referrer=searchResults

What is the slope of a line parallel to the line whose equation is 2x - 5y = 30. Fully simplify your answer.

Answers

Answer: [tex]\boxed{y=\frac{2}{5} x-7}\ as\ an\ example[/tex]

Step-by-step explanation:

Find the slope-intercept form:

[tex]2x-5y=30\\\\-5y=-2x+30\\\\y=\frac{-2x+30}{-5} \\\\y=\frac{2}{5} x-6[/tex]

Therefore the slope is 2/5

An example of a line parallel to this line is

[tex]y=\frac{2}{5} x-7[/tex]

Find the smallest value of k if 360k is a perfect square

Answers

Answer:

k = 1

Step-by-step explanation:

since 360 = 60² , a perfect square , then

360 × 1 ← is a perfect square

that is 360k with k = 1 is a perfect square

10x-15=15x+50 ise x kaçtır?

Answers

benzer terimleri her iki tarafta birleştirmek:

10x = 15x + 65

-5x = 65

x'i izole etmek:

65/-5 = -13

x = -13

Bu yardımcı olur umarım! :)
10x-15=15x+50
Combine like terms
10x-15x=50+15
-5x=65
Divide both sides by -5
x=13

5000 nails in five boxes. the first and second boxes have 2700 nails all together. the second and the third boxes have 2000 nails all together. the third and fourth boxes have 1800 nails all together. the fourth and the fifth boxes have 1700 nails all together. how many nails are in each box

Answers

The number of nails in each box is: 1300, 1400, 600, 1200, and 500

Given 5000 nails in five boxes.

Let X denote the universal set.

Then n(X) = 5000.

Let A denote the first box,

B denote the second box,

C denote the third box,

D denote the fourth box, and

E denotes the fifth box.

The first and second boxes have 2700 nails altogether.

⇒n(A ∪ B) = 2700

The second and third boxes have 2000 nails altogether.

⇒n(B ∪ C) = 2000

The third and fourth boxes have 1800 nails altogether.

⇒n(C ∪ D) = 1800

The fourth and fifth boxes have 1700 nails altogether.

⇒n(D ∪ E) = 1700

We need to find out how many nails are there in each box.

That is to find out: n(A), n(B), n(C), n(D), and n(E).

Note that all these five sets are disjoint. This means that the intersection is empty.

n(A ∪ B ∪ C ∪ D) = n(A ∪ B) + n(C ∪ D) = 2700 + 1800 = 4500

n(E) = n(X) - n(A ∪ B ∪ C ∪ D) = 5000 - 4500 = 500

n(D) = n(D ∪ E) - n(E) = 1700 - 500 = 1200

n(C) = n(C ∪ D) - n(D) = 1800 - 1200 = 600

n(B) = n(B U C) - n(C) = 2000 - 600 = 1400

n(A) = n(A U B) - n(B) = 2700 - 1400 = 1300

Therefore, the number of nails in each box is 1300, 1400, 600, 1200, and 500.

Learn more at:

https://brainly.com/question/13424866

#SPJ9

[tex] \boxed{\begin{gathered} \rm Let \: \psi_1 : [0, \infty ) \to \mathbb{R} , \psi_2 : [0, \infty ) \to \mathbb{R},f :[0, \infty )\to \mathbb{R} \: and \: g :[0, \infty) \to \mathbb{R} \: be \\ \rm functions \: such \: that \: f(0) = g(0) = 0, \\\\ \rm \psi_{1}(x) = {e}^{ - x} + x, \: \: x \geq0, \\ \rm \psi_{2}(x) = {x}^{2} - 2x - 2 {e}^{ - x} + 2, \: \: x > 0, \\ \rm f(x) = \int_{ - x}^{x} ( |t| - {t}^{2} ) {e}^{ - {t}^{2} } \: dt, \: \: x > 0 \\\\ \rm g(x) = \int_0^{ {x}^{2} } \sqrt{t} \: {e}^{ - t} \: dt, \: \: x > 0 \end{gathered}}[/tex]

Which of the following is True?
[tex] \rm (A) \: \rm \: f ( \sqrt{ \ln 3 } )+ g( \sqrt{ \ln3} ) = \dfrac{1}{3} [/tex]
(B) For every x>1, there exists an α ∈ (1,x) such that ψ₁(x)=1+ax

(C) For every x>0, there exists a β ∈ (0,x) such that ψ₂(x)=2x(ψ₁(β)-1)

(D) f is an increasing function on the interval [tex] \bigg [0 , \dfrac{3}{2} \bigg][/tex] ​

Answers

(A) is false. By symmetry,

[tex]\displaystyle f(x) = \int_{-x}^x (|t|_t^2) e^{-t^2} \, dt = 2 \int_0^x (t-t^2) e^{-t^2} \, dt[/tex]

where [tex]|t|=t[/tex] since [tex]x>0[/tex]. Substitute [tex]s=t^2[/tex] to get the equivalent integral,

[tex]\displaystyle f(x) = \int_0^{x^2} (1 - \sqrt s) e^{-s} \, ds[/tex]

Then

[tex]\displaystyle f(x) + g(x) = \int_0^{x^2} e^{-s} \, ds[/tex]

[tex]\displaystyle f(\sqrt{\ln(3)}) + g(\sqrt{\ln(3)}) = \int_0^{\ln(3)} e^{-s} \, ds = \frac23 \neq \frac13[/tex]

(B) is false. Note that [tex]1+\alpha x[/tex] is linear so its derivative is the constant [tex]\alpha[/tex] at every point. We then have

[tex]{\psi_1}'(\alpha) = -e^{-\alpha}+1 = \alpha \implies 1-\alpha = e^{-\alpha}[/tex]

But this has no solutions, since the left side is negative for [tex]\alpha>1[/tex] and the right side is positive for all [tex]\alpha[/tex].

(C) is true. By the same reasoning as in (B), the line [tex]2x(\psi_1(\beta)-1)[/tex] has constant derivative, [tex]2\psi_1(\beta)-2 = 2e^{-\beta+2\beta-2[/tex]. Then

[tex]{\psi_2}'(\beta) = 2\beta-2+2e^{-\beta} = 2e^{-\beta}+2\beta-2[/tex]

holds for all values of [tex]\beta[/tex].

(D) is false. We use the first derivative test. By the fundamental theorem of calculus,

[tex]\displaystyle f(x) = 2 \int_0^x (t-t^2)e^{-t^2}\,dt \implies f'(x) = 2(x-x^2)e^{-x^2}[/tex]

Solve for the critical points.

[tex]f'(x) = 0 \implies x-x^2 = 0 \implies x = 0 \text{ or } x = 1[/tex]

[tex]e^{-x^2}>0[/tex] for all [tex]x[/tex], so the sign of [tex]f'[/tex] depends on the sign of [tex]x-x^2[/tex]. It's easy to see [tex]f'>0[/tex] for [tex]x\in(0,1)[/tex] and [tex]f'<0[/tex] for [tex]x\in\left(0,\frac32\right)[/tex]

PLEASE HELP 4/z>3; z=2

Answers

answer and steps to get it in the picture

(3, 4, 5, ...} is finite or infinite

Answers

The given set is (3, 4, 5, ...} is infinite set.

A set with an infinite number of elements is one that cannot be numbered. A set that has no last element is said to be endless. A set that can be put into a one-to-one correspondence with a suitable subset of itself is said to be infinite. No issue with the in-class assignment.

The stars in the clear night sky, water droplets, and the billions of cells in the human body are just a few examples of endless sets of objects that surround us. A set of natural numbers, however, serves as the best illustration of an infinite set in mathematics. There is no limit to the amount of natural numbers.

Learn more about infinite here

https://brainly.com/question/3928632

#SPJ9

Convert the following temperatures from Fahrenheit to Celsius or vice versa.
C= F-32/1.8
F = 1.8C+32
a. 50°F
b. 75°C
c. -40°C

Answers

Answer:

10°C

Step-by-step explanation:

c = [tex]\frac{F - 32}{1.8}[/tex]  Plug in 50 for F and solve for C

c = [tex]\frac{50-32}{1.8}[/tex]

C = [tex]\frac{18}{1.8}[/tex] Divide

c = 10

P is the point (2, 3) and Q is the point (9, 5).
(a) Find the equation of the line joining PQ.
(b) Find the coordinates of the point where the line PQ intersects the x-axis.
(c) The line y = 5 is the line of symmetry of △PQR. Find the coordinates of R.
(d) Calculate the area of △PQR.
(e) Calculate the length of PQ and hence calculate the perpendicular distance from R to the line PQ.

Answers

We get that the equation of the line PQ will be 2 x - 7 y + 17 = 0.

A) We know that the equation of the line is:

y - y₁ = m (x - x₁)

where m = y₂ - y₁ / x₂ - x₁

Now, we have:

P = ( x₁, y₁) = (2, 3)

Q = (x₂, y₂) = (9, 5)

We get m by substituting the values as:

m = 5 - 3 / 9 - 2 = 2 / 7

The equation then will be:

y - y₁ = 2 / 7 (x - x₁)

Substituting the values, we get that:

y - 3 = 2/7 ( x - 2)

7 y - 21 = 2 x - 4

2 x - 7 y - 4 + 21 = 0

2 x - 7 y + 17 = 0

B) When PQ will intersect x-axis, then the point will be = ( x, 0)

2 x - 7 y + 17 = 0

2 x - 7 (0) + 17 = 0

2 x = -17

x = - 17 / 2

Point = (-17 / 2 , 0)

Therefore, we get that the equation of the line PQ will be 2 x - 7 y + 17 = 0.

Learn more about equation here:

https://brainly.com/question/1214333

#SPJ9

A thermometer in Grand Forks, North Dakota, reads -4.5 °F in January.
The temperature on the same day in February has the same absolute value
as the temperature in January but is not the same temperature. What is
the temperature in February?

Answers

According to this the temperature in February = 4.5°F.

What do you mean by absolute value:

The absolute value (or modulus) of a real number x is its non-negative value regardless of its sign. For example, 5 has an absolute value of 5, and 5 has an absolute value of 5. A number's absolute value can be conceived of as its proximity from zero along the real number line.

Briefing:

The temperature on the same day in February has the same absolute vale as the temperature in the January, but is not the same temperature according to this the temperature in February = 4.5°F.

According to the data:

The absolute value is the same but not the same temperature:

|-4.5| = 4.5

So,

Its 4.5°F

According to this the temperature in February = 4.5°F.

To know more about absolute value visit:

https://brainly.com/question/26497616

#SPJ9

The perimeter of the pentagon below is 63 units. Find the length of side AB.
Write your answer without variables.

Answers

Answer:

15

Step-by-step explanation:

The sides add to 63, so:

[tex]11+3x+11+x+2+2x+3=63 \\ \\ 6x+27=63 \\ \\ 6x=36 \\ \\ x=6 \\ \\ \implies AB=2(6)+3=15[/tex]

Helppp!

h(x)=x^2-6

Evaluate h(-3)

Answers

Answer:

3

Step-by-step explanation:

input -3 into h(x)=x^2-6

h(-3)=-3^2-6=9-6=3

The sum of two numbers is -316. One number is 94 less than the other. Find the numbers

Answers

Answer: -111 and -205

Explanation:

Using the description, we can set up equations to solve for the two numbers. First we assign variables to the numbers: Number 1 will be “x” and number 2 will be “y”.

For the first part [The sum of two numbers is -316] the equation will be:
x + y = -316

For the second part [One number is 94 less than the other] the equation will be:
x = y - 94

Those two equations give us a system of equations we can use to solve for the variables.

Equation 1: x + y = -316
Equation 2: x = y - 94

Since we have an equation for what x is (equation 2), we can substitute that into equation 1 and solve for y.

x + y = -316
(y - 94) + y = -316
[combine the ys]
2y - 94 = -316
[add 94 to both sides]
2y = -222
[divide by 2]
y = -111

Plug y into one of the equations and solve for x and you get: x = -205

Determine if the following relations represent a function
1. {(-1, -2), (0, -2), (1, -2), (2, -2)}
2. {(1, 0), (1,1), (1, 2), (1, -2)}

Answers

1) Yes, because each value of x maps onto only one value of y.

2) No, because the x-value of 1 maps onto 4 different y values.

Answer:

yes, a function.no, not a function.

Step-by-step explanation:

You want to know if the given sets of ordered pairs represent a function.

Function

A function is a relation that maps an input value to a single output value. On a graph, this means no two points are vertically aligned. For a table or set of ordered pairs, it means no input (x) value is repeated.

1.

The input (x) values are {-1, 0, 1, 2} with no repeats. This relation is a function.

2.

The input values are {1, 1, 1, 1} with repeats. This relation is not a function.

Round 1803.2684 to the nearest thousandth

Answers

Answer:

1,803.268

Step-by-step explanation:

so thousandths place is three number after the decimal just see the fourth number after the decimal and if it 5 or greater round the (in this case) 8 but since the fourth number was 4 it stays the same but only three numbers

1803.268
The third number after the decimal is the thousandth place meaning you need to round the 8. Since the last number is under 5 you round down

If we are asked to to find the x-intercept of the graph given its equation, we substitute y = 0 into the equation, because:

The x-intercept is the point where the graph intersects the x-axis and hence the y-coordinate of that point will be 0

The y-intercept will be 0

The x-intercept is the point where the graph intersects the y-axis and hence the y-coordinate of that point will be 0

The x-intercept is always at (0, 0)

Answers

Answer:

1st one is the right option

could someone please help answer this.​

Answers

Answer:

C.) Equally as distant.

Help me ASP SHOW UR WORK THANK YOU!!

Answers

The value of x is 4 , 12 , - 1 resp. in eq 1 , 2 and 3 .

Finding values of unknown variable x .

Following are linear equations in one variable .

On solving them we get ,

1 )          2 ( 3 x - 5 ) = - 4 x + 30

             6 x - 10 = - 4 x + 30

              10 x =40

                 x = 4

2 )          5 x - 6 - 3 x = 18

              2 x = 18 + 6

              2 x = 24

               x = 12

3 )         - 11 - 5 x = 6 ( 5 x + 4 )

               - 11 - 5 x = 30 x + 24

                 - 35 x = 35

                x = - 1

To learn more on linear equation in one variable follow link :

https://brainly.com/question/14323743

#SPJ9

need help on 11-17 pls. Thank u

Answers

11) 58 degrees for both angles
12) 72 and 108 degrees (left to right)
14) b=-3/2
I don’t have the rest I’m sorry
Other Questions
By what specific measurement is construction used as an economic indicator for the u.s. economy? What is the probability that a randomly chosen male respondent or his partner has blue eyes?. How do I solve this problem?On Friday, 4/7 of Parkden School chose pasta for lunch, 1/5 chose pizza and the rest chose salad. What fraction of the pupils chose salad? Every cereal box has a gift inside, but you cannot tell from the outside what the gift is. The store manager assures you that 19 of the 59 boxes on the shelf have the secret decoder ring. The other 40 boxes on the shelf have a different gift inside. If you randomly select two boxes of cereal from the shelf to purchase, what is the probability that both of them have the secret decoder ring? In what ways was english policy towards natives different from those of france and spain? When performing a ____________________ and the f2 generations yield significantly different results, it is likely that the trait is sex linked. A hypothetical futures contract on a nondividend-paying stock with a current spot price of $100 has a maturity of one year. If the T-bill rate is 5%, what should be the future price be? Please help due in less than an hour What cellular component is responsible for energy production and metabolic processes? Is there a difference between [tex]-\frac{x}{y}[/tex] and [tex]\frac{-x}{-y}[/tex] ?Do they mean different things? Which statement provides the information needed to better understand the direct quotation it contains?Excited about what has occurred, Elias Boudinot takes time to reflect, writing, The laws of the State are declared by the highest judicial tribunal in the Country null and void (2).Stand Watie and Elias Boudinot were brothers, and in one letter from Boudinot to Watie, Boudinot explains, The laws of the State are declared by the highest judicial tribunal in the Country null and void (2).Elias Boudinot shares the happy news, telling him, The laws of the State are declared by the highest judicial tribunal in the Country null and void (2).In a letter to Stand Watie, Elias Boudinot expresses his excitement about the Supreme Court ruling against Georgia, stating, The laws of the State are declared by the highest judicial tribunal in the Country null and void (2). a body of mass 7.5kg is moving with the velocity of 1.2, if a force of 15newton is applied to the body calculate its velocity after 2 seconds What is the proper order for how a bill passes through the House? In meiosis, how does prophase i differ from prophase ii (choose all that apply)? How many distinguishable permutations can be formed from the letters in the word sassafras Which equation best represents the graph shown below? Explain in detail how you arrived at your answer by stating each of the mathematical transformations necessary to produce the graph.please I need help 7. Alicia and Dexter are each walking on a straight path. For a particular -second window of time, each has their velocity (in feet per second) measured and recorded as a function of time. Their respective velocity functions are plotted in .Figure 1.4.14. The velocity functions and for Alicia and Damon, respectively.Determine formulas for both and .What is the value and meaning of the slope of ? Write a complete sentence to explain and be sure to include units in your response.What is the value and meaning of the average rate of change of on the interval ? Write a complete sentence to explain and be sure to include units in your response.Is there ever a time when Alicia and Damon are walking at the same velocity? If yes, determine both the time and velocity; if not, explain why.Is is possible to determine if there is ever a time when Alicia and Damon are located at the same place on the path? If yes, determine the time and location; if not, explain why not enough information is provided. Which elements of hpi are met in this statement? patient complains of headache and blurry vision for the past 3 days. Exercise 1 Write frag. next to each sentence fragment. Write s next to each complete sentence.In 1917, OKeeffe had her first art show at the New York gallery owned by Stieglitz. The note-taking system suggested by the text has three columns with headings for main ideas, support material, and?